dog water ur bad kid get better

Answers

Answer 1

don't understand your question


Related Questions

Find the volume of the pentagonal prism?

please help me

Answers

Answer:

126 cm^3

Step-by-step explanation:

Use volume formula (Bh=V): 21×6=126

The volume of the pentagonal prism is 126 cm³

What is pentagonal prism?

Pentagonal prism has two pentagonal bases, top and bottom and five rectangular sides.

How to find the the volume of pentagonal?

Volume of the pentagonal prism = Area of the pentagon• height of the prism

Given:

Area of the pentagon = 21 cm²

Height of the prism = 6 cm

The volume of pentagonal prism = Area of the pentagon * Height of the prism

The volume of pentagonal prism = 21 cm² × 6 cm = 126 cm³

Therefore, the volume of pentagonal prism is 126 cm³

Learn more about the volume of pentagonal prism here:

https://brainly.com/question/31028600

#SPJ6

Find the value of X (in this picture)​

Answers

Answer:

x = 89

Step-by-step explanation:

The sum of the angles of a triangle add to 180 degrees

x+32+ 59 = 180

Combine like terms

x + 91 = 180

Subtract 91 from each side

x +91-91= 180-91

x = 89

NEED HELP ASAP
Which equation does the graph of the systems of equations solve?

two linear functions intersecting at 3, negative 2

−one thirdx + 3 = x − 1
one thirdx − 3 = −x + 1
−one thirdx + 3 = −x − 1
one thirdx + 3 = x − 1

Answers

Answer:

[tex]\frac{1}{3}x-3=-x+1[/tex]

Step-by-step explanation:

Since there are plenty of lines that can pass through a single point, he only way to solve this question is by substituting values of [tex]3[/tex] into each equation and seeing if both equations return a value of [tex]-2[/tex].

Starting with the first answer choice:

[tex]-\frac{1}{3}(3)+3=3-1,\\-1+3=3-1,\\2=2[/tex]

Since none of the equations here return a value of [tex]-2[/tex], the correct answer must not include [tex]-\frac{1}{3}x+3[/tex] or [tex]x-1[/tex].

Thus, we can eliminate answer choices A, C, and D, hence the correct answer is [tex]\boxed{\text{B. }\frac{1}{3}x-3=-x+1}[/tex]

PLEASE HELP!!!

WILL MARK BRAINLIEST!!!

Solve for X.
Multiple choice!

Thank you!

Answers

Answer:

9

Step-by-step explanation:

the 2 angles create a right angle ( indicated by the little square.)

right angles have a measure of 90 degrees

hence, 90 = 72 + 2x

( note that we just created an equation that we can use to solve for x )

we now solve for x using the equation we created.

90 = 72 + 2x

step 1 subtract 72 from each side

90 - 72 = 18

72 - 72 cancels out

we now have 18 = 2x

step 2 divide each side by 2

18 / 2 = 9

2x / 2 = x

we're left with x = 9


Francis borrowed $20 from his dad in the morning. Later, he gave his dad 8
dollars back. What rational number represents the overall amount of money Francis still owes his dad?

Answers

Answer:

13

Step-by-step explanation:

Francis borrowed 20$ and gives his dad 8$ he owes 12$ I hope it helps :)

The answer would be $12 still owed to his father.

120+12314543-900-90+12

Answers

Answer:

12313685 is the answer............happy to help u

12,313,685 !!! happy
to help

The number of points Marcus scored in different rounds of a computer board
game is shown 236, 342, 328, 352, 352, 306, 317
What score does Marcus need in his next game to have a mean of exactly 325?

HELP FAST PLZ

Answers

Answer:583

Step-by-step explanation:

I took the test

i need help with this geometry question pleasee

Answers

Answer:i dont know

Step-by-step explanation:lol

Answer:

tmabm queria saber

Step-by-step explanation:

need answer !! please

Answers

Step-by-step explanation:

-5(4)+2

-20+2

-18

just put 4 on the place of x

Answer:

[tex]=-18[/tex]

Step-by-step explanation:

[tex]g(x)=-5x+2[/tex]

Let's substitute 4 for x and solve.

[tex]g(4)=-5(4)+2[/tex]

[tex]g(4)=-20+2[/tex]

[tex]g(4)=-18[/tex]

This means that when the function of [tex]g[/tex] is 4, then the [tex]x[/tex] value is [tex]-18[/tex]

Hope this helps.

Cary calculated the surface area of a box in the shape of a rectangular prism. She wrote the equation 148 = 2 (6w + 6h + hw) to represent the width and height of the box. She solved for w and got w = StartFraction 74 minus 6 h Over h + 6 EndFraction Which of the following is an equivalent equation?

Answers

Answer:

the answer is w = 148-12h/12+2h

The equivalent equation is -

[tex]$w=\frac{74-6h}{h+6}[/tex]

We have the equation written by Carly → 148 = 2 (6w + 6h + hw) that represent the width and height of the box in the shape of a rectangular prism.

We have to solve for w.

What do you mean by Equivalent expression ?

Any expression written in a form different from the original form, but gives same result for any input are called equivalent expressions.

Solve for x :  [tex]$log(\frac{x}{\omega}) = 2\pi[/tex]

We have -  

[tex]$log(\frac{x}{\omega}) = 2\pi[/tex]

log(x) - log(ω) = 2π

log(x) = 2π + log(ω)

x = [tex]e^{(2\pi + log(\omega))} = e^{2\pi } \times e^{log(\omega)}[/tex]

According to the question, we have -

148 = 2 (6w + 6h + hw)

(6w + 6h + hw) = 74

hw + 6w = 74  - 6h

w(h + 6) = 74 - 6h

[tex]$w=\frac{74-6h}{h+6}[/tex]

Hence, the equivalent equation is -

[tex]$w=\frac{74-6h}{h+6}[/tex]

To solve more questions on Rearranging expression, visit the link below-

https://brainly.com/question/1824488

#SPJ2

Select the correct answer. Which is the simplified form of the expression ? A. B. C. D.

Answers

Answer:

where is the question

Step-by-step explanation:

send question

Answer:

The answer is B: 67/10x + 9

Step-by-step explanation:

I did the test

What are the solutions to the quadratic equation below


Answers

Answer:  B is the correct answer.


what is the probability that 1 or 2 are rolled on a number cube with sides numbered 1, 2, 3, 4,
5, and 6?

Answers

Answer: [tex]\frac{1}{3}[/tex]

Step-by-step explanation:

total sides = 6

p (rolling a 1) = [tex]\frac{1}{6}[/tex]

p (rolling a 2) = [tex]\frac{1}{6\\}[/tex]

Note:

or - add

and - multiply

∴ [tex]\frac{1}{6} +\frac{1}{6} = \frac{2}{6}[/tex]

∴ [tex]\frac{2}{6} = \frac{1}{3}[/tex]

hence, p (rolling a 1 or 2) = [tex]\frac{1}{3}[/tex]

A large soda bottle cap has a radius of 1+2 /2 centimeters. What is the area of the bottle cap? (R the area of a circle is no where r is the radius). 1 + 2v2 A. (5-12 - square IT square centimeters​

Answers

Answer:

bottle top

Step-by-step explanation:

Which fraction is equal to 3?
0A) 30/10
OB) 1/3
OC) 10/3
OD) 13/1​

Answers

Answer:

The answer is 30/10

Step-by-step explanation:

30 divided by 10 is 3.

Answer: A) 30/10

30/10=3

Explanation: Divide 30 by 10 and solve. You could insert this into your calculator to solve. You should get the answer of 3.

BET YOU CANT SOLVE THIS....

Answers

the perimeter of figure EFGH is 10
Beth can’t man sorry

What is the volume of a sphere with a diameter of 57.1 cm, rounded to the nearest tenth of a cubic centimeter?

Answers

Answer:

so we have to use the formula 4/3PiR^3

which if we do we get the volume of

V≈97478.08

Answer:

V≈97478.08cm³

Step-by-step explanation:

Using the formulas

V=4/3πr3

d=2r

V=1/6πd3

D=1

1/6·πd3 1/6π ·57.13≈97478.07565cm³

I hope this helps. I worked hard on this one.

P(5)=581,000e^-0.022t

Answers

Answer:

52,047.96

Step-by-step explanation:

10t+[tex]\geq[/tex]130+3.5t

Answers

Answer:

The answer is [tex]t\geq 20[/tex].

Step-by-step explanation:

To solve the inequality, start by solving for the variable [tex]t[/tex].

To solve for the variable [tex]t[/tex], subtract [tex]3.5t[/tex] from both sides. The inequality will look like [tex]6.5t\geq 130[/tex].

Then, divide both sides by 6.5 in order to get the variable [tex]t[/tex] by itself. The inequality answer will look like [tex]t\geq 20[/tex].

Consider the following sets of sample data: A: 20,347, 20,327, 22,117, 21,762, 20,864, 20,102, 21,684, 20,063, 21,728, 21,580, 21,720, 20,920, 21,442, 20,766 B: 3.38, 4.64, 4.09, 3.93, 4.25, 4.63, 4.78, 4.25 Which of the above sets of sample data has the larger spread

Answers

Answer:

Data B

Step-by-step explanation:

Given the data :

A: 20347, 20327, 22117, 21762, 20864, 20102, 21684, 20063, 21728, 21580, 21720, 20920, 21442, 20766

B: 3.38, 4.64, 4.09, 3.93, 4.25, 4.63, 4.78, 4.25

The spread of a data gives the variation in the data values of a given sample.

To obtain which data has the larger spread, we obtain the coefficient of variation. Which is the ratio of the standard deviation and the mean of the dataset.

(Standard deviation / mean) * 100%

Using calculator :

Data A :

Mean, x = 21101.5714

Standard deviation, s = 700.28925

Coefficient of Variation :

(700.28925 / 21101.5714) * 100% = 3.32%

Data B :

Mean, x = 4.24375

Standard deviation, s = 0.457006955

Coefficient of Variation :

(0.457006955 / 4.24375) * 100% = 10.77%

10.77% > 3.32%

Hence. Data B has a larger spread

Find the length of the third side if necessary right in simplest radical form

Answers

Answer:

[tex] \large{ \tt{❃ \: SOLUTION}} : [/tex]

The longest side , which is the opposite of side of right angle is the hypotenuse ( h ). There are two other sides, the perpendicular ( p ) and the base ( b ) .

In given right triangle , hypotenuse ( h ) = [tex] \sqrt{61} [/tex] , perpendicular ( p ) = 5 & base ( b ) = ?

[tex] \large{ \tt{✻ \: USING \: PYTHAGOREAN\: THEOREM : }}[/tex]

[tex] \large{ \tt{❁ {h}^{2} = {p}^{2} + {b}^{2} }}[/tex]

Plug the values and simplify!

[tex] \large{ ↦( \sqrt{61} })^{2} = {5}^{2} + {b}^{2} [/tex]

[tex] \large{↦ \: 61 = 25 + {b}^{2} }[/tex]

[tex] \large{↦25 + {b}^{2} = 61 }[/tex]

[tex] \large{↦ {b}^{2} = 61 - 25}[/tex]

[tex] \large{↦ {b}^{2} = 36}[/tex]

[tex] \large{↦ {b} = \sqrt{36} }[/tex]

[tex] \large{↦ b = \sqrt{ \underline{3 \times 3} \times \underline{ 2 \times 2} }}[/tex]

[tex] \large{↦b = 3 \times 2}[/tex]

[tex] \large{ \boxed{ \boxed{ \bold{↦b = 6 \: units }}}}[/tex]

Hence , the length of a third side is [tex] \boxed{ \tt{6 }}[/tex] units .

✺ Never give up on something that you actually want !

۵Hope I helped! ツ

☼ Have a wonderful day / night ! ☃

# StayInAndExplore ! ☂

▁▁▁▁▁▁▁▁▁▁▁▁▁▁▁▁▁▁▁▁▁▁▁▁▁

___________________________________

Problem:

Find the length of the third side if necessary right in simplest radical form.

Formula:

[tex]\quad\quad\quad\quad\boxed{\tt{ {c}^{2} = \sqrt{ {a}^{2} + {b}^{2} }}} [/tex]

Remember:

a = perpendicular b = base c = hypotenuse

Given:

[tex]\quad\quad\quad\quad\tt{ a = 5 } [/tex]

[tex]\quad\quad\quad\quad\tt{ b = ? } [/tex]

[tex]\quad\quad\quad\quad\tt{ {c = \sqrt{61} }} [/tex]

Solution:

[tex]\quad\quad\quad\quad\tt{ {( \sqrt{61}) }^{2} = \sqrt{ {(5)}^{2} + {b}^{2} }} [/tex]

[tex]\quad\quad\quad\quad\tt{ {61}= \sqrt{ {25} + {b}^{2} }} [/tex]

[tex]\quad\quad\quad\quad\tt{ 61 = \sqrt{ 25 + {b}}} [/tex]

Let's convert the "b" like this.

[tex]\quad\quad\quad\quad\tt{ {b } = \sqrt{25 - 61 }} [/tex]

[tex]\quad\quad\quad\quad\tt{ {b } = \sqrt{36 }}[/tex]

[tex]\quad\quad\quad\quad\tt{ {b } = 6 }[/tex]

So the final answer is:

[tex]\quad\quad\quad\quad \boxed {\boxed{\tt{ \color{magenta} {b } = 6\:units }}}[/tex]

___________________________________

#CarryOnLearning

✍︎ C.Rose❀

I WILL MARK BRAINLIEST!


Two identical rubber balls are dropped from different heights. Ball 1 is dropped from
a height of 110 feet, and ball 2 is dropped from a height of 276 feet. Use the
function f(t) -16t2 + h to determine the current height, f(t), of a ball dropped from a
height h, over given time t.
Write a function for the height of ball 1.
hi(t)

Answers

9514 1404 393

Answer:

  h₁(t) = -16t² +110

Step-by-step explanation:

Put the given initial height into the given formula. That will give the requested function. If the function name is supposed to be h₁(t), then rename it.

  f(t) = -16t² +110 . . . . . . h = 110, the initial height

  h₁(t) = -16t² +110

► Find the percent change and tell whether it is a percent increase or a
percent decrease.

1 Original amount: 20
End amount: 15

2 Original amount: 30
End amount: 45

3 Original amount: 625
End amount: 550

4 Original amount: 320
End amount: 112

5 Original amount: 165
End amount: 222.75

6 Original amount: 326
End amount: 423.80

7 Original amount: 27
End amount: 38.61

8 Original amount: 60
End amount: 70.02

Answers

Answer:

1 Original amount: 20

End amount: 15

(20 - 15)/20 x 100 = (5/20) x 100 = 25% decrease

2 Original amount: 30

End amount: 45

(45 - 30)/30 x 100 = (15/30) x 100 = 50% increase

3 Original amount: 625

End amount: 550

550/625 =550 ÷ 625 =0.880.88 =0.88 × 100/100 =(0.88 × 100)/100 =88/100 =88% decrease

4 Original amount: 320

End amount: 112

(112-320):320x100 =100 =(112:320-1)x100 =100 =35-100 = -65% decrease

5 Original amount: 165

End amount: 222.75

165/222.75 =165 ÷ 222.75 ≈0.7407407407407410.740740740740741 =0.740740740740741 × 100/100 =(0.740740740740741 × 100)/100 ≈74.074074074074/100 =74.074074074074% ≈74.07% Increase

6 Original amount: 326

End amount: 423.80

(423.8-326):326x100 =100 =(423.8:326-1)x100 =100 =130-100 = 30% Increase

7 Original amount: 27

End amount: 38.61

(38.61-27):27x100 =100 =(38.61:27-1)x100 =100 =143-100 = 43% Increase

8 Original amount: 60

End amount: 70.02

(70.02-60):60x100 =100 =(70.02:60-1)x100 =100 =116.7-100 = 16.7% Increase

please help zkhdusjdushs​

Answers

Answer:

48

Step-by-step explanation:

16×3=48

therefore the answer is forty eight

Add - 3/x + 7y/x .
-4y/2x
-3 + 7y/x
- 10y/x
-3 + 7y/2x

Answers

Answer:

[tex]-\frac{3}{x} + \frac{7y}{x} = \frac{-3+ 7y}{x}[/tex]

Step-by-step explanation:

Given

[tex]-\frac{3}{x} , \frac{7y}{x}[/tex]

Required

Add

The statement can be interpreted as:

[tex]-\frac{3}{x} + \frac{7y}{x}[/tex]

Take LCM

[tex]-\frac{3}{x} + \frac{7y}{x} = \frac{-3+ 7y}{x}[/tex]

What is the value of (–7 + 3i) – (2 – 6i)?
–9 + 9i
–9 – 3i
–5 – 3i
–5 + 9i

Answers

Answer:

- 9 + 9i

Step-by-step explanation:

(- 7 + 3i) - (2 - 6i)

- 7 + 3i - 2 + 6i

- 7 - 2 + 9i

- 9 + 9i

The value of(-7+3i)-(2-6i)
-9+9i

the students in charge of the class booth at a carnival would like to earn $3 for every item they sell. they spent $55 for the materials to make the items. solve the inequality 3x-55_>65 which represents how many items they need to sell to make profit of at least $65

Answers

Answer:

x ≥ 40

Step-by-step explanation:

3x - 55 ≥ 65

combine like terms

3x  ≥ 65 + 55

3x  ≥120

divide both sides of the equation by 3

x  ≥ 40

I need help please I don’t understand

Answers

9514 1404 393

Answer:

  1. ∠EDF = 104°

  2. arc FG = 201°

  3. ∠T = 60°

Step-by-step explanation:

There are a couple of angle relationships that are applicable to these problems.

the angle where chords meet is half the sum of the measures of the intercepted arcsthe angle where secants meet is half the difference of the measures of the intercepted arcs

The first of these applies to the first two problems.

1. ∠EDF = 1/2(arc EF + arc UG)

  ∠EDF = 1/2(147° +61°) = 1/2(208°)

  ∠EDF = 104°

__

2. ∠FHG = 1/2(arc FG + arc ES)

  128° = 1/2(arc FG +55°) . . . substitute given information

  256° = arc FG +55° . . . . . . multiply by 2

  201° = arc FG . . . . . . . . . subtract 55°

__

3. For the purpose of this problem, a tangent is a special case of a secant in which both intersection points with the circle are the same point. The relation for secants still applies.

  ∠T = 1/2(arc FS -arc US)

  ∠T = 1/2(170° -50°) = 1/2(120°)

  ∠T = 60°

The center of a circle is at (6,-7) and the diameter of the circle is 22. Which of the following is the equation of the circle? (Part 1 and Part 2)

Answers

Answer:

P1.H.x²+y²=34

centre[h,k]=(0,0)

point=[3,5]

now

radius=[tex]\sqrt{(0-3)²+(0-5)²}=\sqrt{34}[/tex]

now

equation of a circle is;

(x-h)²+(y-k)²=[tex]\sqrt{34}²[/tex]

x²+y²=34

P:2I.(x-6)²+(y+7)²=121

centre[h,k]=(6,-7)

diameter=22

radius[r]=22/2=11

now

equation of a circle is;

(x-h)²+(y-k)²=r²

(x-6)²+(y+7)²=11²

(x-6)²+(y+7)²=121

Find the equation of the line that
is perpendicular to y = -4x + 3
and contains the point (8, 1).
y =
x+
Enter

Answers

Answer:

Step-by-step explanation:

L : y = -4x + 3

with a slope of -4.

Any line perpendicular to L will have a slope m such that

m ( -4) = -1, or m = 1/4

Any line perpendicular to L passing through point (a,b) will therefore have the form

P : (y-b) = (x-a)/4

When (a,b) = (8,1) as given, then substitute in P to get

P : (y-1) = (x-8)/4

y=x/4 -2 + 1

y = x/4 -1

Other Questions
Which of the following proteins could be used in connective tissues, tendons, or hair? Which of the statements regarding the nitrogen cycle is true? a. Denitrification occurs when a nitrogen containing compound is produced from nitrogen in the soil. b. Atmospheric nitrogen is absorbed through the leaves of plants. c. Nitrogen fixation is the process atmospheric nitrogen is converted into biologically usable forms. d. Animals obtain nitrogen through the process of cellular respiration Can someone help me solv for x? Help please match with the words Evaluate the expression when g=7 and h=32.H+35 over g How many outcomes are in the sample space when rolling a standard number cube and spinning a spinner with 4 equal sections? Whatcha method development was used in this paragraph?A.cause and effect B.example C.comparison and contrast D.sequence of events The relationship between the actual air temperature x (in degrees Fahrenheit) and the temperature y adjust for wind chill (in degrees Fahrenheit, given a 20 mph wind) is given by the following formula:y= -22 + 1.4xEstimate the actual temperature if the temperature adjusted for wind chill is -15 degrees Fahrenheit. short story that's personally written with the same mood Which element has their valence electron farthest from the nucleus?K (Potassium)Li (Lithium)Na (Sodium)Rb (Rubidium) 3. (20 points) You can buy or sell a 3.5% coupon $1,000 par U.S. Treasury Note that matures in 6 years. The first coupon payment pays 6 months from now, and the Note pays coupons semi-annually until maturity. It also pays par on maturity. The Yield to Maturity of the Note right now (treat this as your discount rate) is 3.000%. (a) What are the cash flows associated with this Note Understanding the CaseMatch each action to Marbury or Madison.Refused to honor an appointmentMarburyMadisonWas appointed as a federal judgeSupported the Judiciary Act of 1789Argued for original jurisdiction If p= 2q^3-1, what is the value of q when p is 13a) 6b) 7c) 14/2d) 4393PLS HELP QUICKLY OMG TYSM HELP ME PLEASEEEEEEEE ASAPPPP !! Advantages of relative location for commercial activities find the greatest common factor of 15x^6 and 33x^4y^5 Jom!06 PMWhat is the slope of the line that passes through the points (-4, 4) and (-6, 6)?Write your answer in simplest form. how do you multiply 4.56 x 1,000 Why did the unity of the nation remain in question despite adopting the Constitution?A. It did not allow for amendmentsB. It did not provide a comprehensive plan to followC. It did not create a judicial systemD. It did not set a structure of government What is the oxidation state of each element in the compound CaSO4?